a) Null Hypothesis: H₀: P>0.22
Alternative hypothesis: H₁:P<0.22
b) Test statistic Z = 2.5
The sample size is n = 1100.
Given the information, political research selected 1100 voters from the town and discovered that 25% of the citizens supported the building.
Given sample percentage "p" = 25% = 0.25, this follows.
A campaign strategist decided to check the assertion that more than 22% of people support building based on available data.
Assuming Population Proportion 'P' = 22% = 0.22
Q = 1 - p = 1 - 0.22=0.78
Given the available information, a political strategist wishes to test the assertion that more citizens prefer building than 22%, thus we assume the null hypothesis is true.
Null Hypothesis: H₀: P > 0.22
Alternative hypothesis: H₁: P < 0.22
The test of statistic
Z = (p - P) ÷ √(PQ ÷ n)
Z = (0.25 - 0.22) ÷ √(0.22 × 0.78 ÷ 1100)
Z = 2.5
Learn more about the null hypothesis at
https://brainly.com/question/23693764?referrer=searchResults
#SPJ4
0.006 Reapting in a fraction in simplest form
Answer:
3/500
Step-by-step explanation:
6/1000 ÷2 =
3/500
if zeros are reciprocal of each other then find k. (1-3k)x2 +4x-5
Answer: Let's assume that the zeros of the given quadratic equation are a and 1/a, since they are reciprocals of each other. By the sum-product relationships for zeros of a quadratic equation, we have:
a + 1/a = -b/a1 = -4/(1-3k) (where b = 4 and a1 = 1-3k)
Multiplying both sides by a1, we get:
a1(a + 1/a) = -4
Expanding the left-hand side, we get:
a1a + a1/a = -4
Multiplying both sides by a, we get:
a1a² + a1 = -4a
Substituting a1 = 1-3k, we get:
(1-3k)a² + (1-3k) = -4a
Rearranging, we get:
(1-3k)a² + 4a + (3k-1) = 0
Since a is one of the zeros of the quadratic, we know that the quadratic can be factored as:
(1-3k)(a - r)(a - s) = 0
where r and s are the other two roots of the quadratic. By expanding the left-hand side, we get:
(1-3k)(a² - (r+s)a + rs) = 0
Comparing with the expanded form of the quadratic, we see that:
r + s = -4/(1-3k)
rs = (3k-1)/(1-3k)
By Vieta's formulas, we know that rs = c/a1, where c is the constant term of the quadratic. Substituting c = -5 and a1 = 1-3k, we get:
rs = -5/(1-3k)
Equating this with the expression we obtained earlier for rs, we get:
-5/(1-3k) = (3k-1)/(1-3k)
Multiplying both sides by 1-3k and simplifying, we get:
-5 = (3k-1)(3k-2)
Expanding the right-hand side, we get:
-5 = 9k² - 15k + 2
Simplifying, we get:
9k² - 15k - 7 = 0
Using the quadratic formula, we get:
k = (15 ± sqrt(15² + 497))/18
k = (15 ± sqrt(429))/18
Therefore, the two possible values of k are:
k = (15 + sqrt(429))/18 ≈ 1.57
k = (15 - sqrt(429))/18 ≈ -0.24
Note that both of these values of k satisfy the condition that the zeros of the quadratic are reciprocals of each other.
Step-by-step explanation:
HELP MEEE!! What's the domain and range?
Answer:
domain: [- ∞ , ∞]
range: (8 , -∞]
Step-by-step explanation:
- ∞ < x < ∞
y < 8
Please help!! If you can.
Answer:
4 is the answer is this equation
Graph the line with the equation y = 4/5x + 2
Answer: I have provided a screenshot of what your graph should look like so uh yeah you got this!
The graph of the line with the equation y = 4/5x + 2 is given in below attachment.
To graph the line with the equation y = (4/5)x + 2, we can use the slope-intercept form, which is y = mx + b.
where "m" represents the slope and "b" represents the y-intercept.
In this case, the slope (m) is 4/5, and the y-intercept (b) is 2.
To plot the line, we can start by plotting the y-intercept, which is the point (0, 2).
From there, we can use the slope to find additional points and connect them to form the line.
Let's plot a few more points:
When x = 5:
y = (4/5)(5) + 2
= 4 + 2 = 6
So we have the point (5, 6).
When x = -5:
y = (4/5)(-5) + 2
= -4 + 2
= -2
So we have the point (-5, -2).
To learn more on slope intercept form click:
https://brainly.com/question/9682526
#SPJ6
h(t) = t + 2 + 3; Find h(6)
h(6) = 11
Step-by-step explanation:Hi there !
h(t) = t + 2 + 3
= t + 5
replace t = 6
h(6) = 6 + 5
= 11
Good luck !
Please help em fast I really need help
lf ƒ(x) = 2(x + 1)² and g(x) = 3x- 2 determine f[g(2)]
Answer:
f(g(2)) = 50
Step-by-step explanation:
evaluate g(2) then substitute the result obtained into f(x)
g(2) = 3(2) - 2 = 6 - 2 = 4 , then
f(4) = 2(4 + 1)² = 2(5)² = 2(25) = 50
Answer:
f[g(2)] = 50
Step-by-step explanation:
Given functions:
\(f(x)=2(x+1)^2\)
\(g(x)=3x-2\)
We are asked to determine f[g(2)], which is known as a composite function. When solving composite functions, you always work inside out.
Step 1: Find the value of g(2) by substituting 2 for x in function g(x).
\(\implies g(2)=3(2)-2\)
\(\\\implies g(2)=6-2\Rightarrow g(2)=\boxed{4}\)
Step 2: Substitute the found value into the composite function.
\(\\\implies f[g(2)]= 2(4+1)^2\)
\(\implies f[g(2)] = 2(5)^2 \Rightarrow 2(25) \Rightarrow \boxed{50}\)
Hence, the value of the composite function f[g(2)] is 50.
Learn more about composite functions here:
brainly.com/question/28062427
brainly.com/question/26215408
What information would verify that LMNO is an
isosceles trapezoid? Check all that apply.
a. LN MO
b. LM ON
c. LO = MN
d. ZL = ZN
e. ZLEM
f. IRR
The correct answers are options b. LM ≅ ON and d. ∠L ≅ ∠N as they verify that LMNO is an isosceles trapezoid.
To verify that LMNO is an isosceles trapezoid, we need to check if the following conditions are met:
1. One pair of opposite sides must be parallel (base angles are equal).
2. The non-parallel sides (legs) are congruent.
From the given options:
a. LN ≅ MO - This does not verify LMNO as an isosceles trapezoid, as it only shows congruence between the parallel sides.
b. LM ≅ ON - This verifies LMNO as an isosceles trapezoid, as it shows that the non-parallel sides (legs) are congruent.
c. LO ≅ MN - This does not verify LMNO as an isosceles trapezoid, as it only shows congruence between the diagonals.
d. ∠L ≅ ∠N - This verifies LMNO as an isosceles trapezoid, as it shows that the base angles are congruent, indicating that the opposite sides are parallel.
e. ∠L ≅ ∠M - This does not verify LMNO as an isosceles trapezoid, as it only shows congruence between adjacent angles.
Thus, options b and d are the correct answers that verify LMNO is an isosceles trapezoid.
Learn more about trapezoid here:
https://brainly.com/question/14458711
#SPJ11
Ejemplo: Martin necesita sembrar unos terrenos de cultivo. Ha gestionado un
préstamo del Banco "A" de S/ 5000 para pagar en tres años con una tasa de
interés del 4% mensual, ¿qué cantidad de interés pagará?
Datos: C= S/ 5000 t= 3 años r= 4% (mensual) = ?
8
Answer:
Step-by-step explanation:
10
How long will an initial bank deposit of $10,000 grow to $23,750 at 5% annual compound interest?
Given:
P = $10000
r= 5%
Find:
we have to find the time for which initial bank deposit of $10,000 grow to $23,750 at 5% annual compound interest.
Explanation:
The formula for compound interest is
First, convert R as a percent to r as a decimal
r = R/100
r = 5/100
r = 0.05 per year,
Then, solve the equation for t
t = ln(A/P) / n[ln(1 + r/n)]
t = ln(23,750.00/10,000.00) / ( 1 × [ln(1 + 0.05/1)] )
t = ln(23,750.00/10,000.00) / ( 1 × [ln(1 + 0.05)] )
t = 17.729 years
Summary:
The time required to get a total amount of $23,750.00 with compounded interest on a principal of $10,000.00 at an interest rate of 5% per year and compounded 1 times per year is 17.729 years.
(about 17 years 9 months)
use analytic methods to find (a) the local ex- trema, (b) the intervals on which the function is increasing, and (c) the intervals on which the function is decreasing
Use analytic methods,
(a) The local extrema,
1) If f'(x) > 0 for all x on (a , c) and f'(x)<0 for all x on (c , b), then f(c) is a local maximum value.
2) If f'(x) < 0 for all x on (a , c) and f'(x)>0 for all x on (c , b), then f(c) is a local maximum value.
(b) The intervals on which the function is increasing
Write properties of function :
Increasing interval : ( - ∞ , 0 )
(c) The intervals on which the function is decreasing
Write properties of function :
Decreasing interval : ( 0 , ∞ )
Given that,
Use analytic methods (a) the local extrema, (b) the intervals on which the function is increasing, and (c) the intervals, then the function is,
A function's growing (or decreasing) periods match the periods when its derivative is positive (or negative). As a result, we can easily determine the intervals where a function increases or decreases by taking its derivative and analyzing it to determine if it is positive or negative.
(a) How do we find the local extrema?Let f be continuous on an open interval (a , b) that contains a critical x-value.
1) If f'(x) > 0 for all x on (a , c) and f'(x)<0 for all x on (c , b), then f(c) is a local maximum value.
2) If f'(x) < 0 for all x on (a , c) and f'(x)>0 for all x on (c , b), then f(c) is a local maximum value.
(b) The intervals on which the function is increasing
Write properties of function :
Increasing interval : ( - ∞ , 0 )
(c) The intervals on which the function is decreasing
Write properties of function :
Decreasing interval : ( 0 , ∞ )
Therefore,
Use analytic methods,
(a) The local extrema,
1) If f'(x) > 0 for all x on (a , c) and f'(x)<0 for all x on (c , b), then f(c) is a local maximum value.
2) If f'(x) < 0 for all x on (a , c) and f'(x)>0 for all x on (c , b), then f(c) is a local maximum value.
(b) The intervals on which the function is increasing
Write properties of function :
Increasing interval : ( - ∞ , 0 )
(c) The intervals on which the function is decreasing
Write properties of function :
Decreasing interval : ( 0 , ∞ )
To learn more about Analytic methods visit :
brainly.com/question/19463434
#SPJ4
I NEED HELP GUYS PLEASE I CANT FAIL AGAIN A translation was applied to square ABCD to form
A'B'C'D.
Which line segment defines the translation?
SHE
ОАС!
y
5
OBD
А
B
4.
O C'B
3
O DA
2-
D
A'
C
B'
-5-4-3-
2
3
4
5
X
D
C
No trolls or links or i will report
Answer:
92.4 ft²Step-by-step explanation:
Total surface area comprises one rectangular base and two pairs of diferent triangles.
Area of rectangle
A = lwArea of triangle
A = 1/2bhSubstitute and find
A = 7*4 + 2(1/2*4*6.3) + 2(1/2*7*5.6) = 92.4 ft²Area of base rectangle
7(4)28ft²Area of rest triangles
2(1/2BH)+2(1/2bh)BH+bh4(6.3)+7(5.6)64.56ft²Now
TSA
64.56+2892.56ft²help me..........................
Answer:
d.90l
Step-by-step explanation:
Question 4 of 10
Which of the following could be the ratio between the lengths of the two legs
of a 30-60-90 triangle?
Check all that apply.
□A. √2:√2
B. 15
□ C. √√√√5
□ D. 12
DE √3:3
OF. √2:√5
←PREVIOUS
SUBMIT
The ratios that could be the lengths of the two legs in a 30-60-90 triangle are √3:3 (option E) and 12√3 (option D).
In a 30-60-90 triangle, the angles are in the ratio of 1:2:3. The sides of this triangle are in a specific ratio that is consistent for all triangles with these angles. Let's analyze the given options to determine which ones could be the ratio between the lengths of the two legs.
A. √2:√2
The ratio √2:√2 simplifies to 1:1, which is not the correct ratio for a 30-60-90 triangle. Therefore, option A is not applicable.
B. 15
This is a specific value and not a ratio. Therefore, option B is not applicable.
C. √√√√5
The expression √√√√5 is not a well-defined mathematical operation. Therefore, option C is not applicable.
D. 12√3
This is the correct ratio for a 30-60-90 triangle. The ratio of the longer leg to the shorter leg is √3:1, which simplifies to √3:3. Therefore, option D is applicable.
E. √3:3
This is the correct ratio for a 30-60-90 triangle. The ratio of the longer leg to the shorter leg is √3:1, which is equivalent to √3:3. Therefore, option E is applicable.
F. √2:√5
This ratio does not match the ratio of the sides in a 30-60-90 triangle. Therefore, option F is not applicable. So, the correct option is D. 1 √2.
For more such questions on lengths
https://brainly.com/question/28322552
#SPJ8
The table below shows that the number of miles driven by Jamal is directly proportional to the number of gallons he used.
Gallons Used
Gallons Used
Miles Driven
Miles Driven
14
14
525
525
43
43
1612.5
1612.5
47
47
1762.5
1762.5
How many gallons of gas would he need to travel
296.25
296.25 miles
Jamal would need approximately 7.9 gallons of gas to travel 296.25 miles.
We can use the concept of direct variation to solve this problem. Direct variation means that two quantities are related by a constant ratio. In this case, the number of miles driven is directly proportional to the number of gallons used.
To find the constant of proportionality, we can use the given data. From the table, we can see that when Jamal used 14 gallons, he drove 525 miles. So we can write:
14/525 = k
where k is the constant of proportionality.
Solving for k, we get:
k = 14/525
Now we can use this value of k to find how many gallons Jamal would need to travel 296.25 miles. Let x be the number of gallons he would need. Then we can write:
x/296.25 = k
Substituting the value of k, we get:
x/296.25 = 14/525
Solving for x, we get:
x = (296.25 × 14) / 525
x ≈ 7.9
To learn more about proportionality click on,
https://brainly.com/question/29005048
#SPJ1
For what value of x is the equations 2^2x+7 = 2^15 true?
Answer:
Step-by-step explanation:
To find the value of x, solve the equation. Do this by rearranging the equation to make x the subject.
Answer:
x=32761 /4
Step-by-step explanation:
Step 1: Simplify both sides of the equation.
4x+7=32768
Step 2: Subtract 7 from both sides.
4x+7−7=32768−7
4x=32761
Step 3: Divide both sides by 4.
What is 85 degrees Fahrenheit in Celsius?
Answer:
29.4444
Step-by-step explanation:
Find the x and y intercept of y=-3x+4
Answer:
I've attached the answer below.
Step-by-step explanation:
did it tell you what x and y equal
for what values of x in [0,2π] does the graph of f(x)=x+2sinx have a horizontal tangent?
the values of x in [0, 2π] where the graph of f(x) has a horizontal tangent are x = 2π/3, x = 4π/3, and x = π.
To find the values of x in [0, 2π] where the graph of f(x) = x + 2sin(x) has a horizontal tangent, we need to find where the derivative of the function is zero or undefined.
The derivative of f(x) is:
f'(x) = 1 + 2cos(x)
For the derivative to be zero, we need:
1 + 2cos(x) = 0
Solving for cos(x), we get:
cos(x) = -1/2
This is true when x = 2π/3 or x = 4π/3.
Now we need to check if the derivative is undefined at any point in the interval [0, 2π]. The derivative is undefined when cos(x) = -1, which occurs at x = π.
To know more about derivative visit:
brainly.com/question/29144258
#SPJ11
Which equation represents the graphed function?
A)-3x + 2 = y
B) -2/3x+2=y
C)3/2x-3=y
D)2x-3=y
Answer:
(C) 3/2x-3=yStep-by-step explanation:
The point on the x-axis is 2
The point on the y - axis is -3
#3) Find the equation of the linear relationship represented in the table below.
x
у
-7.
5
-3
1
1
-3
5
-7
Brandon is not smiling. He just left the
dentist's office, and he has 5 cavities! If
it takes the dentist 14 minutes to fill one
cavity, how long will it take to fill all 5 of
Brandon's cavities?
Answer:
it should take 70 mins to fill all 5
hope this helps!! :)
Step-by-step explanation:
14 x 5 = 70
Answer:
70
Step-by-step explanation:
14 minutes = 1 cavity
x minutes = 5 cavities
70= x
Find the slope of the line graphed below.
Answer:
m = 3/4
Step-by-step explanation:
Start in (- 2,1) up three and four to the right. Positive slope. (Going uphill positive +, and downhill is negative - )
i need help with this
which term describes the percent of voters casting ballots?
The term which is used for describing the number of voters who casted their votes using the ballots is called as voter turnout.
The process of selecting a candidate for a suitable post who takes care of the policies which will be framed for the people is called as voting. In this process, the members/ citizens casts their vote in the favor of their suitable candidate. Many times voters are not valid and their votes are considered as null and void.
The voter turnout is either the percentage of registered voters, eligible voters, or all voting-age people ( this varies from place to place depending upon the minimum voting age required). The voter turnout is also referred to the number of people who go to it or take part in it. High voter turnout is generally considered a sign of a healthy democracy.
Learn more about voter turnout at:
brainly.com/question/29639449
#SPJ4
One angle of a parallelogram measures 37°. What are the measures of the other three angles in the parallelogram?
The measures of the other two angles in the parallelogram are both 143 degrees.
what is a parallelogram?A parallelogram is a quadrilateral with two sides that are parallel to one another. A parallelogram has equal-length opposite sides and equal-length opposite angles. The internal angles that are extra to the transversal on the same side. A parallelogram's total interior angles add up to 360 degrees.
A parallelogram is formed by the junction of two parallel lines, and its opposite angles are equal.
Then, there are two 35 degree angles. Two further angles that are equivalent are noted: A parallelogram's angles add up to 360 degrees. Following that, x+x+37+37=360
2x=360-74
2x=286
x=143.
To know more about parallelogram visit:
brainly.com/question/29147156
#SPJ1
 HELP ASAP TEST IS OVER SOON!!! PLEASEEE
A television screen is a rectangle 35 inches wide and 20 inches long. A television size is advertised as the length of its diagonal. What is the size of the television? In other words, what is the length of the diagonal? Around the answer to the nearest hundredth.
Answer:
35 in
Step-by-step explanation:
Give an example of a function that is a dilation and a reflection of the parent function.
Answer: f(x) = -1/2(x+3)
Example: g(x) = -1/2(-x-3)
This function is a dilation and a reflection of the parent function f(x) because when graphed, the two functions have the same shape but are reflected over the y-axis and are half the size of the original.
Matthew wants to take out a loan to buy a car. He calculates that he can make repayments of $35,000 per year. If he can get a six-year loan with an interest rate of 9.25%, what is the maximum price he can pay for the car?
The maximum price Matthew can pay for the car, considering his repayment capability and the loan terms, is approximately $126,318.29.
To determine the maximum price Matthew can pay for the car, we need to consider his repayment capability and the terms of the loan.
Matthew can make annual repayments of $35,000. Since the loan term is six years, the total amount he can repay over the loan period is $35,000 multiplied by six, which equals $210,000.
To calculate the maximum price of the car, we need to account for the interest rate of 9.25%. The interest rate represents the cost of borrowing and is applied to the loan amount.
Let's assume the loan amount is denoted by P.
The formula to calculate the future value of a loan with interest is:
FV = P(1 + r)^n
Where:
FV = Future value (total amount repaid)
P = Principal amount (maximum price of the car)
r = Interest rate per period (9.25% or 0.0925)
n = Number of periods (six years)
Since Matthew can repay a total of $210,000 over the loan period, we can set up the equation:
$210,000 = P(1 + 0.0925)^6
Now we can solve for P:
P = $210,000 / (1 + 0.0925)^6
Evaluating this expression, we find:
P ≈ $126,318.29
Therefore, the maximum price Matthew can pay for the car, considering his repayment capability and the loan terms, is approximately $126,318.29.
Learn more about capability here
https://brainly.com/question/31154332
#SPJ11